Inéquation $x^y + y^x > 1$ — Les-mathematiques.net The most powerful custom community solution in the world

Inéquation $x^y + y^x > 1$

Bonjour, cela fait quelque jour que j'essaie de résoudre cette équation pour tout réel x, y > 0.

J'ai montré que le minimum de f(x) = x^x est atteint pour x = 1/e mais je ne vois pas comment l'appliquer sur ce problème. J'ai essayé pleins de choses comme remplacer x^y par e^yln(x) et cette piste :
x^y+y^x = (y * x/y)^y + (x * y/x)^x = y^y * (x/y)^y + x^x * (y/x)^x.
En revanche, je n'arrive pas à conclure... Auriez-vous des idées ? Merci d'avance.

Réponses

  • Suppose $0<x \leq y<1 $ SPDG, étudie $x \mapsto g_y(x)=x^y+y^x$. En fin du fil je donnerai la référence de ce problème.
  • Comme l'a dit Chaurien pose $0<x\leq y<1$ et pose $x=cos(a)^2$ et $y=sin(a)^2$ en oubliant pas la relation $cos(x)^2+sin(x)^2=1$ tu as normalement une inégalité à une variable en oubliant pas non plus que la condition imposée $x+y=1$ est optimale

    En espérant ne pas avoir d'énormités !

    Edit:Gerard0 c'est corrigé...(:D
  • Merci pour vos réponses, je vais essayer de suivre vos pistes.

    En revanche, je ne suis pas sur de comprendre ce que tu veux dire Bloy.noel. Cette propriété existe-t-elle ? :

    $\forall(x,y)\in\mathbb{R}^{+2}, \exists a\in\mathbb{R}, x = cos^2(a)$ et $y = sin^2(a)$

    Sinon, je ne vois pas du tout ce que tu veux dire.
  • Désolé je suis allez un peu vite :
    Euh non ton affirmation est fausse je pense que tu as inversé(e) les quantificateurs "il existe" et "quelque soit"
  • Tu es sympa Errys,

    Bloy.noel pose x=cos²(x) !!!
    Écrit comme il l'est actuellement, c'est du n'importe quoi.

    Cordialement.
  • Bonjour @Errys,

    La relation est vraie pour tout x et y deux réels strictement positifs.
    On pose $f(x)=x^y+y^x$, cette fonction est dérivable. La dérivée vaut...

    Pour $y\geq 1$, la dérivée est positive, la fonction croissante et elle tend vers 1 en 0.
    Pour $0<y<1$ :
    Si $x\geq 1$, alors $x^y >1$. Et donc la relation est vérifiée.
    Si $0<x<1$, alors la derivee s’annule lorsque ... et on prend le logarithme et on étudie la fonction $g(x)=\ln y + (y-1) \ln x -\ln(-\ln y)- x \ln y$. La dérivée vaut... et s’annule en $(y-1)/\ln y \geq 1$. La fonction g est monotone décroissante. L’equation $g(x)=0$ possède une solution unique.

    La fonction f est alors croissante puis décroissante. Elle vaut 1 en 0 à la limite, et vaut >1 en 1 quelque soit la position de l’optimum par rapport à 1. Ceci termine la démonstration.
  • Merci beaucoup pour ton aide !

    Je vais essayer de remplir les pointillés même si je bloque au bout d'un moment :

    $f(x) = x^y+y^x=x^y+e^{xln(y)}$

    Soit : $f'(x)=yx^{y-1} + ln(y)e^{xln(y)}=yx^{y-1}+ln(y)y^x$

    $f'(x)=0$

    $\iff yx^{y-1} + y^xln(y)=0$

    $\iff yx^{y-1}=-y^xln(y)$

    $\iff \dfrac{yx^{y-1}}{y^x}=-ln(y)$

    $\iff \dfrac{x^{y-1}}{y^{x-1}}=-ln(y)$

    $\iff x^{y-1}y^{1-x}=-ln(y)$

    $\iff e^{ln(x)(y-1)}e^{ln(y)(1-x)}=-ln(y)$

    $\iff e^{ln(x)(y-1)+ln(y)(1-x)}=-ln(y)$

    $\iff ln(x)(y-1)+ln(y)(1-x)=ln(-ln(y))$

    $\iff ln(x)(y-1)+ln(y)(1-x)-ln(-ln(y))=0$

    On pose donc $g(x) = ln(x)(y-1)+ln(y)(1-x)-ln(-ln(y))$

    $g'(x) = \dfrac{y-1}{x} -ln(y)$

    Or, $0<y<1$ donc $ln(y)<0$ soit $-ln(y)>0$
    De plus, $\dfrac{y-1}{x}<0$ car $-1<y-1<0$

    Donc $g$ est strictement décroissante, on a :
    $\lim\limits_{x\to0}\Big(ln(x)(y-1)+ln(y)(1-x)-ln(-ln(y))\Big)=+\infty$
    $\lim\limits_{x\to1}\Big(ln(x)(y-1)+ln(y)(1-x)-ln(-ln(y)) = -ln(-ln(y))\Big)$

    De plus, $0<y<1$ donc $=-\infty<ln(y)<0$ soit : $0<-ln(y)<+\infty$ et : $-\infty<ln(-ln(y))<+\infty$ soit : $-\infty<-ln(-ln(y)) < +\infty$

    Donc selon la valeur de $y$, $g(x)=0$ peut admettre une solution ou non.. Je vais m'arreter là pour ce soir, je reprendrai demain ! J'éspère ne pas avoir fais de coquille, je fais tout ca sur LateX et étant en terminale, je ne suis pas habitué à ces démonstrations à rallonges.
  • @ Errys
    Bravo pour ton LaTeX. Pense au \ devant ln pour faire $\ln$, c'est plus joli.
    Tu n'es pas obligé de reproduire tous les détails de tes calculs. Donne juste les étapes importantes. Tu gagneras du temps pour toi et tu n'encourras pas le risque de lasser ton lecteur.
    Bon courage.
    Fr. Ch.
  • Bonjour,

    Une erreur (de ma part) pour le sens de variation de la fonction $g$. On trouve, pour $0<x<1, 0<y<1$, $g'(x) = {y-1 \over x} - \ln y$ et alors $g'(x) = 0 \implies x={y-1 \over \ln y} \leq 1.$ Cette dernière inégalité est due à la relation $\forall t \in \R, t >0, \ln t \leq t-1.$
    La fonction $g$ est donc strictement décroissante puis croissante sur $]0,1[$ avec $g(1) = - \ln (-\ln y)$ donc le signe dépend de la position de $y$ par rapport à $1/e$...

    L'étude semble plus compliquée que mon premier poste. Mais c'est une voie possible de résolution.

    Idée : on peut peut-être simplifier grâce à la symétrie entre $x$ et $y$... avec $y \geq x.$
  • Je pense avoir un preuve plutôt simple :
    On part de l'inégalité suivante avec $0<a\leq b\leq 1$ et un $n$ que je préciserai à la fin:
    $$a^b\geq \frac{a^{\frac{1}{n}}}{(a+b)^{\frac{1}{n}}}$$
    On passe au logarithme on a alors :
    $b\ln(a)\geq \frac{1}{n}\ln(a)-\frac{1}{n}\ln(a+b)$
    Ou encore :
    $n\ln(a)\geq \frac{\ln(a)-\ln(a+b)}{b}$
    On reconnaît un accroissement fini a droite on applique donc l'inégalité des accroissements finis en vérifiant bien qu'on est sur un intervalle fermée ($[a;b]$) dérivable sur ($]a;b[$) et que la fonction dérivée (ici $\frac{1}{x}$) est bien bornée (ici par $\frac{1}{a}$) on obtient donc :
    $n\ln(a)\geq\frac{1}{a} \ge \frac{\ln(a)-\ln(a+b)}{b}$
    Pour que l'inégalité soit valide il suffit donc de prendre $n\ge \frac{1}{a\ln(a)}$
    Du reste par symétrie on a donc :
    $$a^b+b^a\geq \frac{a^{\frac{1}{n}}}{(a+b)^{\frac{1}{n}}}+\frac{b^{\frac{1}{n}}}{(a+b)^{\frac{1}{n}}}$$
    Avec $n\ge \max(\frac{1}{a\ln(a)};\frac{1}{b\ln(b)})$
    Enfin il faut prouver qu'on a bien :
    $$a^{\frac{1}{n}}+b^{\frac{1}{n}}\geq (a+b)^{\frac{1}{n}}$$
    Ce qui réécrit ainsi :
    $$(a^{\frac{1}{n}}+b^{\frac{1}{n}})^n\geq (a+b)$$
    N'est autre que l'inégalité de Minkowski inverse
    Source :
    https://proofwiki.org/wiki/Minkowski's_Inequality_for_Sums/Index_Less_than_1
    http://math.univ-lyon1.fr/capes/IMG/pdf/new.IAF.pdf
    Bien cordialement.
    Ps:Je réfléchis à un preuve plus simple utilisant l'inégalité de Young .Et si j'ai écrit trop de bêtises répondez moi vite .
  • Concours général...de je ne sais plus quelle année.
  • Lien inactif...
  • Il me semble avoir vu passer sur ce forum une autre inégalité concernant $x^y+y^x$ mais plus difficile. Quelqu'un s'en souvient-il ?
  • Bonjour,

    On a étudié les équations, pour deux réels, $x^y=y^x$ et $x^x=y^y$
  • @ YvesM
    Merci pour l'aide, mais ce n'est pas ça.
    Tu évoques deux équations dans $\mathbb Q_+^*$.
    Moi je pense à une inégalité dans $\mathbb R_+^*$, comme $x^y+y^x>x+y$ ou $x^y+y^x>x^x+y^y $, je ne me souviens plus.
    Ce n'était pas aussi récent.
    Bonne journée.
    Fr. Ch.
Connectez-vous ou Inscrivez-vous pour répondre.
Success message!